¿Puede la mecánica cuántica ser realmente lo mismo que la teoría determinista subyacente?

Estoy perplejo por los artículos recientes de 't Hooft que dan una construcción explícita para una teoría determinista subyacente basada en números enteros que es indistinguible de la mecánica cuántica en escalas experimentalmente accesibles. ¿Significa que es una complejidad determinista disfrazada de aleatoriedad cuántica?

http://arxiv.org/abs/1204.4926

Sobre el mismo tema: Discreción y Determinismo en Supercuerdas arxiv.org/abs/arXiv:1207.3612

Respuestas (4)

Creo que al menos algunos lectores ya deberían haber notado que muchos de estos argumentos, particularmente los más patéticos, son cuestiones de redacción más que de física. Una vez que haya hecho su modelo lo suficientemente simple, puede mapear cualquier cosa en cualquier cosa. Este fue mi punto de partida: si un sistema es lo suficientemente trivial, puedes hacer lo que quieras. Ahora, ¿cómo podemos generalizar posteriormente algunos resultados tan simples en algo más interesante?

Esta ha sido la regla básica de mi enfoque. No estoy interesado en absoluto en los teoremas "no-go", estoy interesado en la pregunta "¿qué se puede hacer en su lugar?" Admito que no puedo resolver los problemas del universo, no he encontrado la Teoría del Todo. En lugar de anunciar patéticamente lo que no debes hacer, trato de construir modelos, paso a paso.

Ahora creo que he producido algunos modelos que vale la pena discutir. Quizás aún no sean lo suficientemente grandes y complicados para describir nuestro universo, pero pueden plantear nuestras preguntas sobre las distinciones entre la mecánica cuántica y las teorías clásicas en una nueva perspectiva. Claramente, si un sistema es demasiado simple, esta distinción desaparece. Pero, ¿hasta dónde puede llegar uno? Recuerde que los autómatas celulares pueden llegar a ser tremendamente complejos, y los modelos mecánicos cuánticos también. ¿Hasta dónde podemos llegar relacionando los dos? Así es como debes mirar mis papeles. Creo que la pregunta es muy importante, y se puede ir mucho más allá de lo que algunas personas quieren que creamos al relacionar los modelos cuánticos con los clásicos.

¿Y es un cálculo incorrecto si a alguien no le gusta la redacción?

Estoy de acuerdo en que los modelos son interesantes y el programa es importante, pero el principal problema es que no has demostrado que la mecánica cuántica en una base beable sea equivalente a la estadística clásica en un CA, y esto podría no ser cierto (creo que es no es cierto sin más restricciones). Haré una pregunta directa, ya que sus interesantes respuestas y comentarios han permitido agudizar el pensamiento sobre esto.

La sabiduría actual (experimental y teórica) sobre los enfoques deterministas del no determinismo cuántico simplemente dice que cualquier teoría determinista que subyace a la mecánica cuántica debe ser no local. Luego, la investigación continúa discutiendo la naturaleza precisa de esta no localidad o descartando ciertas versiones.

Por otro lado, están aquellos que construyen teorías deterministas no locales que de alguna manera se reducen a QM. Se dedica mucho trabajo a la mecánica de Bohm, que, sin embargo, tiene dificultades para recuperar una teoría cuántica de campos realista.

El artículo de t'Hooft persigue un enfoque diferente, basado en la discreción. Sin embargo, sus resultados actualmente son muy limitados, simplemente reproduciendo el oscilador armónico.

Tu primera frase puede ser demasiado categórica. Por ejemplo, tengo entendido que no ha habido evidencia experimental libre de lagunas de no localidad. Por otro lado, la no localidad es una afirmación extraordinaria, por lo que necesita una prueba extraordinaria.
t'Hooft no "reproduce" nada; tiene un espacio lineal que es un espacio de estado cuántico, simplemente trunca la evolución para que sea determinista sobre una base especial. La dinámica real sigue siendo cuántica, no es realmente un autómata --- no puedes imaginar que los estados son clásicos debajo.
"Sin embargo, sus resultados son actualmente muy limitados, solo reproducen el oscilador armónico" . org/abs/1207.3612 y arxiv.org/abs/1205.4107 ) Sin embargo, los argumentos allí me parecen ser una extensión del argumento del primer artículo. No pude encontrar ninguna escapatoria en el mapeo... de ahí la pregunta... ¿qué es exactamente lo que viola el esquema?
Estoy de acuerdo en que no hay muchos resultados, especialmente algo que distinga al modelo de los demás. Muchas gracias por su comentario. -- Ceniza
@a25bedc5-3d09-41b8-82fb-ea6c353d75ae: (Elija un mejor nombre para la identificación) - No he mirado los detalles porque mi impresión sobre todos estos intentos es que son callejones sin salida; muchos tecnicismos, pero en el fondo nada que agregue comprensión a lo que ya sabemos. Si el indeterminismo cuántico se genera en la Naturaleza a partir de una dinámica clásica discreta subyacente, apuesto a que es una forma elegante, al igual que la mecánica cuántica es una variación elegante de la mecánica clásica, no desordenada.
@akhmeteli: la no localidad de partículas es un hecho experimental ordinario (ya visible en experimentos de interferencia); es sólo la forma precisa que toma en teoría lo que está en duda. Así que no estoy de acuerdo con que necesite una verificación extraordinaria. Los problemas filosóficos desaparecen una vez que uno abandona la imagen de las partículas; entonces no queda nada contrario a la intuición que necesite una atención extraordinaria.
@Arnold Neumaier: ¿Esta "no localidad de partículas" es una cuestión de su opinión personal o podría dar una referencia? Permítanme señalar que la interferencia de partículas de tipo cuántico se demostró en un sistema clásico: PRL 97, 154101 (2006) (p. ej., hekla.ipgp.fr/IMG/pdf/Couder-Fort_PRL_2006.pdf )
@akhmeteli: La óptica geométrica es la vista de partículas clásica de la luz clásica, y tiene todas las características no locales que aparecen en la mecánica cuántica, una vez que vas más allá del ámbito de la validez de la aproximación. Esto me dice (aunque no es el punto de vista principal) que la raíz de la no localidad cuántica es tratar los objetos cuánticos como partículas en lugar de excitaciones de campo. En el contexto de la teoría cuántica relativista de campos, todo es local, lo que demuestra que los aspectos extraños no son más que una mala elección de visualización intuitiva.
@Arnold Neumaier: no puedo aceptar la óptica geométrica como evidencia de "no localidad de partículas"; podría mencionar la ley de Coulomb con el mismo "éxito"; sí, no es local, pero es solo una aproximación, que falla en los experimentos exactamente donde predice la no localidad. Por lo tanto, no estamos de acuerdo sobre si la no localidad es una afirmación extraordinaria, y usted no proporcionó ninguna referencia para respaldar su punto de vista. Además, insisto en que hasta ahora no hay evidencia experimental positiva de no localidad en la teoría cuántica debido a lagunas.
@akhmeteli: Toda la física es solo una aproximación, que falla si describe la situación con mayor detalle. Y no hay evidencia experimental positiva de nada microscópico si uno está buscando lagunas tan cuidadosamente como en la literatura sobre la no localidad.
Localidad=>Determinismo (ver JS Bell en "Sesenta y dos años de incertidumbre").
@Arnold Neumaier: Claro:-) Entonces, ¿por qué se publican artículos experimentales sobre la eliminación de lagunas en las mejores revistas en este momento, si las lagunas son tan insignificantes como usted describe? ¿Por qué una carrera tan febril entre los mejores experimentadores tratando de cerrar las lagunas? Zeilinger y sus coautores escribieron recientemente: "Violar la desigualdad de Bell y al mismo tiempo cerrar todas esas lagunas es uno de los desafíos más importantes que aún están abiertos en la física fundamental en la actualidad". (PNAS 16 de noviembre de 2010 vol. 107 no. 46 19708-19713). Por eso dije que tu resumen puede ser demasiado categórico :-)
@quantropy: ''Locality=>Determinism'' solo bajo algunos supuestos, que no se cumplen en la teoría cuántica de campos.
@akhmeteli: Uno nunca puede cerrar todas las lagunas. Eso está en la naturaleza del trabajo experimental.
@Arnold Neumaier: No proporciona referencias, y no estoy seguro de que esta sea una opinión general, por ejemplo, Zeilinger parece no estar de acuerdo con ella en este caso particular (lo que, como dije, indica que de hecho fue demasiado categórico al describir la opinión de los expertos de la situación experimental). Bien puede ser que tenga razón en este caso particular, y las violaciones no se pueden demostrar con todas las lagunas cerradas, pero eso significaría que las lagunas son realmente cruciales, en lugar de insignificantes, como parece argumentar.
Nunca dije que todas mis respuestas fueran convencionales. Sin embargo, suelen ser correctos (y los corrijo cuando resultan erróneos). La verdad no depende del número de detractores.

Ciertamente, es posible que QM se base en un mecanismo físico determinista. Los teoremas de no-go como el teorema de Bell o el "teorema del libre albedrío" de Conway y Kochen no son efectivos contra las teorías deterministas de variables ocultas porque requieren el no determinismo como uno de sus supuestos. Todavía hay muchos físicos que afirman que el determinismo ha sido refutado pero están cometiendo la falacia lógica. Sin embargo, es demasiado pronto para decir si 't Hooft está en el camino correcto.

-1: Esto es incorrecto. El teorema de Bell es efectivo contra las teorías de variables ocultas deterministas que son locales. El argumento del superdeterminismo es ridículo, no es ciencia, es teoría de la conspiración.

Los papeles de t'Hooft no son válidos. Cometen un error, que es que asumen que solo porque el operador de evolución de tiempo discreto en un sistema cuántico es una permutación en alguna base, la teoría cuántica es entonces una teoría clásica.

t'Hooft considera sistemas cuánticos de tiempo discreto en los que la evolución temporal es, en cierto modo, una permutación discreta. Entonces, si tiene un sistema de 3 estados, permuta 1 a 2 a 3. Luego analiza el espacio de todas las superposiciones de estos tres estados y descubre que puede recuperar la mecánica cuántica. Luego declara que "la mecánica cuántica es equivalente a un sistema determinista clásico".

Esto esta simplemente mal. Supongo que t'Hooft está pensando que si comienzas en algún estado base, te quedas en un estado base para siempre, simplemente permutando el estado base y, por lo tanto, debe ser un sistema determinista clásico. Pero el punto es que el espacio de estados incluye todo tipo de superposiciones cuánticas de los estados base, y estos otros estados , los estados no base, son superposiciones no por probabilidad clásica, sino por amplitudes cuánticas.

Si tienes amplitudes cuánticas, incluso si los estados básicos evolucionan por permutación, la teoría obviamente puede reproducir la mecánica cuántica, porque es mecánica cuántica.

De hecho, aquí hay un teorema: dado cualquier hamiltoniano mecánico cuántico de dimensión finita H, existe un sistema de permutación que incluye este hamiltoniano en una aproximación, actuando sobre un subespacio de los estados.

La prueba: diagonalizar H a una matriz diagonal N por N con valores propios N, y aproximar las energías N mediante números racionales con denominadores primos enormes, pags i / q i 1 < i < norte , y toma un paso de tiempo unitario. Multiplique todos los q_i juntos y llame al producto Q. Luego, la exponencial de t por el hamitloniano es periódica con un período Q pasos de tiempo.

Considere ahora un espacio de estado cuya base está etiquetada por una tupla de N números enteros de 1 a Q. Deje que la permutación hamiltoniana tome el elemento base (a_1,....a_n) para a i a i + s i dónde s i es el producto de todas las q excepto q_i, y la Z q i inverso multiplicativo de pags i . Esta permutación hamiltoniana tiene la propiedad de que sus valores propios incluyen un subconjunto con pags i / q i . Proyéctese a este subespacio y llámelo su sistema cuántico.

Este proceso, o cualquier cosa que se le parezca, no puede llamarse "sistema determinista" de ninguna manera. Todavía hay estados que son superposiciones. Si tiene un sistema clásico verdadero, el estado se describe mediante una distribución de probabilidad en el estado inicial desconocido, no mediante amplitudes de probabilidad para superposiciones del estado actual desconocido. En el momento en que describe los estados por superposiciones, no está extrayendo la mecánica cuántica, la está introduciendo.

Esta es la razón por la que t'Hooft puede derivar resultados matemáticos que son mecánicos cuánticos, está usando la mecánica cuántica, pero con la restricción de que se reduce a una permutación sobre una base. Esto no explica por qué vemos superposiciones de giros electrónicos en la naturaleza, no produce estas superposiciones por ignorancia de los valores clásicos, pone las superposiciones a mano.

Me gustan las motivaciones de t'Hooft y admiro su pensamiento independiente, pero esto no es válido. No hace lo que dice que hace. Calificar de engañosa la afirmación de que estos son modelos clásicos es caritativo.

't Hooft cree que existe una base ontológicamente preferida y que las superposiciones de esos estados básicos no son reales. El tratamiento bohmiano de las interacciones de medición muestra que se puede tener una base preferida y aun así describir correctamente otros observables, aunque los bohmianos todavía tienen superposiciones en su ontología, como estados de la onda piloto (a menos que tomen el camino "nomológico" y traten la onda piloto específica de un sistema individual como una ley dinámica en lugar de un estado físico)...
@MitchellPorter: Sé lo que dice , pero esto es ridículo --- tienes que decirme cómo es que un electrón de laboratorio se describe mediante una superposición. No ayuda decir que hay una base en la que el hamiltoniano es una permutación. Si no sabemos en qué elemento base está nuestro universo, lo describimos con una distribución de probabilidad, no con amplitudes. Entonces no hay razón para que el electrón en el laboratorio se describa con amplitudes. Está mal.
En la mecánica de Bohm con una onda piloto específica, puede sustituir la onda piloto específica en las ecuaciones de movimiento para los objetos clásicos de la teoría, y termina con una teoría pseudoclásica en la que una ecuación de movimiento clásica se aumenta con una potencial no local. Debe ser posible explicar la superposición en términos de este potencial no local, porque la teoría sigue siendo idéntica a la mecánica de Bohm con una onda piloto específica, pero nadie ha tomado esta ruta y exhibido cómo se ven tales explicaciones...
En el artículo posterior a este, 't Hooft construye su supuesto mapeo de una CA a una QFT. La CA es realmente simple pero el mapeo es un poco no trivial; al menos, todavía no he captado la esencia de ello. Será difícil decir algo concreto sobre cómo o incluso si 't Hooft puede dar cuenta de las superposiciones observadas, hasta que alguien comprenda esta etapa posterior de su trabajo reciente.
@MitchellPorter: Me tomó mucho tiempo entenderlo, porque claramente está mal, y traté de hacer un verdadero mapa de CA a QM. Lo que está haciendo es lo que describí: toma un sistema QM y lo transforma en un caso en el que se convierte en una permutación sobre una base, y cuando puede hacer esto (que es siempre), declara que ha sacado QM. de un autómata clásico. La declaración es falsa, el método está produciendo un "autómata cuántico t'Hooft", no un autómata clásico, ya que incluye estados de superposición a priori.
La nueva versión v3 de mi artículo quant-ph/ 1405.1548 se envió a ArXiv hoy, 21 de diciembre de 2015. "La interpretación de los autómatas celulares de la mecánica cuántica". Mi respuesta a la pregunta publicada aquí es: sí. Las críticas siempre son bienvenidas. incluso si proviene de aficionados, aunque no puedo garantizar respuesta.